Xem bài viết đơn
Old 25-07-2013, 11:58 AM   #5
hansongkyung
+Thành Viên+
 
Tham gia ngày: Jan 2012
Đến từ: Han Tae Woong - IMO 1998
Bài gởi: 493
Thanks: 109
Thanked 417 Times in 241 Posts
Gửi tin nhắn qua Yahoo chát tới hansongkyung
Trích:
Nguyên văn bởi einstein1996 View Post
Bạn Hùng coi lại xem phải là $FW$ là trục đẳng phương của $\omega_1 ; \omega_2$ như vậy thì ta chỉ có $O_1O_2 \perp FW$ và $HF \perp AW$. Phải chứng minh được A, F, W thảng hàng.
Ta có $AN \cdot AC = AM \cdot AB$ vì $\bigtriangleup AMN \sim \bigtriangleup ABC$ nên $A$ thuộc trục đẳng phương của 2 đường tròn $\omega_1; \omega_2$

Cám ơn bạn, mình sẽ edit lại cho hoàn chỉnh hơn

[RIGHT][I][B]Nguồn: MathScope.ORG[/B][/I][/RIGHT]
 

thay đổi nội dung bởi: hansongkyung, 25-07-2013 lúc 12:03 PM
hansongkyung is offline   Trả Lời Với Trích Dẫn
 
[page compression: 8.79 k/9.92 k (11.33%)]